Search found 109 matches


15^x = 15^y only if x=y. You are not given that and therefore cannot make that assumption. However, you can manipulate the equation to get (15^x)/(15^y) = (4^y)/(4^2) or 15^(x-y) = 4^(y-2). Looking at this equation we see an integer power of an odd number equal to an integer power of an even number...

by ajmoney09

Thu Aug 04, 2011 3:34 pm
Forum: Problem Solving
Topic: Solving For X
Replies: 4
Views: 1288

Can Someone help with this please? The explanation skips a few steps that I don't understand so a nice little walk through will really help here! [15^x + (15^x * 15^1)]/(4^y) = 15^y 15^x[1 + 15] = 15^y * 4^y 15^x * 16 = 15^y * 4^y 15^x * 2^4 = 15^y * 2^(2y) Since the bases are the same on left hand...

by ajmoney09

Thu Aug 04, 2011 10:17 am
Forum: Problem Solving
Topic: Solving For X
Replies: 2
Views: 930

Solving For X

Need Some help please! OA: A

by ajmoney09

Wed Aug 03, 2011 12:18 pm
Forum: Problem Solving
Topic: Solving For X
Replies: 4
Views: 1288

Solving For X

Can Someone help with this please? The explanation skips a few steps that I don't understand so a nice little walk through will really help here!

by ajmoney09

Wed Aug 03, 2011 11:13 am
Forum: Problem Solving
Topic: Solving For X
Replies: 2
Views: 930

gmatblood wrote:D for sure.

Its the problem of the locality and the oppenent is not the nly one in the public of the locality.
Politician's stand is irrelevant
But the passage says nothing about the opponents argument.

by ajmoney09

Sat Jul 23, 2011 6:45 am
Forum: Critical Reasoning
Topic: elections
Replies: 11
Views: 3190

C.

If the politician is going to say "He wants to build these, but he lives here so don't listen to him as he naturally doesn't support that structure. I want to build these." - logically I'd ask where do you live?

D - Don't see how that is irrelevant.

by ajmoney09

Fri Jul 22, 2011 7:59 pm
Forum: Critical Reasoning
Topic: elections
Replies: 11
Views: 3190

IMO C. A - I think the passage means to say that both are equally important and go hand in hand. not that one is more important B - Can't be inferred, the risk is always there and this says "Do not" - strong words. C - I can have the best equipment, but without proper training it won't mat...

by ajmoney09

Fri Jul 22, 2011 7:48 pm
Forum: Critical Reasoning
Topic: Inference-Manhattan CAT
Replies: 10
Views: 1721

E is right.

You can't use "as" because we are comparing.

Example: We had 10 more pies than the clowns.

by ajmoney09

Fri Jul 22, 2011 7:19 pm
Forum: Sentence Correction
Topic: Comparison "As" vs "Than"
Replies: 5
Views: 2673

A person buys a share for $ 50 and sells it for $ 52 after a year. What is the total profit made by him from the share? (I) A company pays annual dividend (II) The rate of dividend is 25% I got stumped (I) INSUFFICIENT - We Don't know how much each dividend was. (II) INSUFFICIENT - We Still Don't k...

by ajmoney09

Sun Apr 10, 2011 7:48 am
Forum: Data Sufficiency
Topic: [DS][share] HSPA posts
Replies: 3
Views: 1131

The sentence makes sense without the ", even when cycling off-road," so pretend that was not there. now read the sentence: The local chapter of the Bike Riders’ Association has repeatedly railed against the municipal authorities, protesting that requiring cyclists to wear bike helmets li...


(2) is Insufficient because the question wants to know what does Y = ? If we plug in -8 into the formula: |3-y|=11 = |3--8|=11 = |3+8|=11 = |11|=11 = 11=11; So why can equal -8 here, but if we plug in 14 we can also get the answer of 11. |3-y|=11 = |3-14|=11 = |-11|=11 = 11=11 I think here you might...

by ajmoney09

Sat Apr 09, 2011 9:45 am
Forum: Data Sufficiency
Topic: Work your abs
Replies: 3
Views: 1133

I get E.

Used plug and chug. But to avoid messy numbers srcc25anu is much faster....But again plug and chug here and go from bottom to top....Would of gotten it on the 1st one this time.

by ajmoney09

Wed Mar 23, 2011 6:17 pm
Forum: Problem Solving
Topic: Tricky Area VIC problem
Replies: 2
Views: 996

Answer is B. I plugged in easy values. a+b = P & a-b=q IF: a=5 b=3 5+3=8 5-3=2 P = 8 Q = 2 a(b)= 5(3) = 15 Plug and chug now.... usually start from bottom to top because on problems like this they want you to waste time, plugging and chugging every where and it is sometimes on the bottom, but yo...

by ajmoney09

Wed Mar 23, 2011 6:05 pm
Forum: Problem Solving
Topic: VIC problem from Knewton
Replies: 5
Views: 1022

I said A.

1 out of ever 4...shoots 4 he is going to hit it once.

by ajmoney09

Wed Mar 23, 2011 5:58 pm
Forum: Problem Solving
Topic: Probability Of Shots
Replies: 7
Views: 1116

If x < 0, then [-x × |x|]^½ is A. -x B. -1 C. 1 D. x E. x^½ http://www.urch.com/forums/gmat-problem-solving as x<0; therefore |x|= -x; so [-x * -x]^1/2; (x^2)^1/2; therefore possible values of expression can be either x, -x; now as x<0; therefore expression yield -x as an answer. hence A Like th...

by ajmoney09

Wed Mar 23, 2011 5:54 pm
Forum: Data Sufficiency
Topic: [-x × |x|]^½ is
Replies: 6
Views: 1843